Asymptotic approximation regarding the Gamma function $Gamma$.












6












$begingroup$


On the wikipedia page for Gamma function I saw an interesting formula
$$
lim_{nto infty} frac{Gamma(n+alpha)}{Gamma(n)n^alpha} = 1
$$

for all $alphainBbb C$. I couldn't find the source of this and searching here in MSE didn't provide the result I want.




Could anyone show me how this formula is derived?




I'm very inexperienced with properties/identities of $Gamma$ so forgive me if this question is trivial.










share|cite|improve this question









$endgroup$












  • $begingroup$
    Use Stirlings Approximation. It's probably on that Wikipedia page.
    $endgroup$
    – Brevan Ellefsen
    Jan 16 at 22:46










  • $begingroup$
    So Stirlings approximation also works in complex case? Thank you, I didn't know that before.
    $endgroup$
    – BigbearZzz
    Jan 16 at 22:47










  • $begingroup$
    This follows from Gautschi's Inequality, which can be proven using the log-convexity of the Gamma function.
    $endgroup$
    – robjohn
    Jan 17 at 4:40


















6












$begingroup$


On the wikipedia page for Gamma function I saw an interesting formula
$$
lim_{nto infty} frac{Gamma(n+alpha)}{Gamma(n)n^alpha} = 1
$$

for all $alphainBbb C$. I couldn't find the source of this and searching here in MSE didn't provide the result I want.




Could anyone show me how this formula is derived?




I'm very inexperienced with properties/identities of $Gamma$ so forgive me if this question is trivial.










share|cite|improve this question









$endgroup$












  • $begingroup$
    Use Stirlings Approximation. It's probably on that Wikipedia page.
    $endgroup$
    – Brevan Ellefsen
    Jan 16 at 22:46










  • $begingroup$
    So Stirlings approximation also works in complex case? Thank you, I didn't know that before.
    $endgroup$
    – BigbearZzz
    Jan 16 at 22:47










  • $begingroup$
    This follows from Gautschi's Inequality, which can be proven using the log-convexity of the Gamma function.
    $endgroup$
    – robjohn
    Jan 17 at 4:40
















6












6








6


1



$begingroup$


On the wikipedia page for Gamma function I saw an interesting formula
$$
lim_{nto infty} frac{Gamma(n+alpha)}{Gamma(n)n^alpha} = 1
$$

for all $alphainBbb C$. I couldn't find the source of this and searching here in MSE didn't provide the result I want.




Could anyone show me how this formula is derived?




I'm very inexperienced with properties/identities of $Gamma$ so forgive me if this question is trivial.










share|cite|improve this question









$endgroup$




On the wikipedia page for Gamma function I saw an interesting formula
$$
lim_{nto infty} frac{Gamma(n+alpha)}{Gamma(n)n^alpha} = 1
$$

for all $alphainBbb C$. I couldn't find the source of this and searching here in MSE didn't provide the result I want.




Could anyone show me how this formula is derived?




I'm very inexperienced with properties/identities of $Gamma$ so forgive me if this question is trivial.







real-analysis number-theory special-functions gamma-function






share|cite|improve this question













share|cite|improve this question











share|cite|improve this question




share|cite|improve this question










asked Jan 16 at 22:12









BigbearZzzBigbearZzz

8,69121652




8,69121652












  • $begingroup$
    Use Stirlings Approximation. It's probably on that Wikipedia page.
    $endgroup$
    – Brevan Ellefsen
    Jan 16 at 22:46










  • $begingroup$
    So Stirlings approximation also works in complex case? Thank you, I didn't know that before.
    $endgroup$
    – BigbearZzz
    Jan 16 at 22:47










  • $begingroup$
    This follows from Gautschi's Inequality, which can be proven using the log-convexity of the Gamma function.
    $endgroup$
    – robjohn
    Jan 17 at 4:40




















  • $begingroup$
    Use Stirlings Approximation. It's probably on that Wikipedia page.
    $endgroup$
    – Brevan Ellefsen
    Jan 16 at 22:46










  • $begingroup$
    So Stirlings approximation also works in complex case? Thank you, I didn't know that before.
    $endgroup$
    – BigbearZzz
    Jan 16 at 22:47










  • $begingroup$
    This follows from Gautschi's Inequality, which can be proven using the log-convexity of the Gamma function.
    $endgroup$
    – robjohn
    Jan 17 at 4:40


















$begingroup$
Use Stirlings Approximation. It's probably on that Wikipedia page.
$endgroup$
– Brevan Ellefsen
Jan 16 at 22:46




$begingroup$
Use Stirlings Approximation. It's probably on that Wikipedia page.
$endgroup$
– Brevan Ellefsen
Jan 16 at 22:46












$begingroup$
So Stirlings approximation also works in complex case? Thank you, I didn't know that before.
$endgroup$
– BigbearZzz
Jan 16 at 22:47




$begingroup$
So Stirlings approximation also works in complex case? Thank you, I didn't know that before.
$endgroup$
– BigbearZzz
Jan 16 at 22:47












$begingroup$
This follows from Gautschi's Inequality, which can be proven using the log-convexity of the Gamma function.
$endgroup$
– robjohn
Jan 17 at 4:40






$begingroup$
This follows from Gautschi's Inequality, which can be proven using the log-convexity of the Gamma function.
$endgroup$
– robjohn
Jan 17 at 4:40












4 Answers
4






active

oldest

votes


















5












$begingroup$

The most usual derivation of this would involve the Stirling-Laplace asymptotic for $Gamma(s)$. I'm mildly surprised that this wasn't explicitly worked out in Wiki, or some other easily accessible places on-line.



In fact, a much simpler approach obtains (a stronger version of) this asymptotic via "Watson's Lemma", which is itself easy to completely prove from simple things, going back over 100 years. In various places in the literature, the lemma is in fact called "the oft-reproven Watson's lemma". :)



The case you mention is a simple corollary of the very first example I wrote out in some notes on asymptotic expansions: http://www.math.umn.edu/~garrett/m/mfms/notes_2013-14/02d_asymptotics_of_integrals.pdf






share|cite|improve this answer











$endgroup$









  • 1




    $begingroup$
    Thank you for you answer. By the way, your link doesn't work for me for some reason.
    $endgroup$
    – BigbearZzz
    Jan 16 at 22:53












  • $begingroup$
    Oop, let me try to repair the link...
    $endgroup$
    – paul garrett
    Jan 16 at 23:03










  • $begingroup$
    I think it works now... :)
    $endgroup$
    – paul garrett
    Jan 16 at 23:04










  • $begingroup$
    Now it also works perfectly for me. Thanks!
    $endgroup$
    – BigbearZzz
    Jan 16 at 23:16



















4












$begingroup$

$newcommand{bbx}[1]{,bbox[15px,border:1px groove navy]{displaystyle{#1}},}
newcommand{braces}[1]{leftlbrace,{#1},rightrbrace}
newcommand{bracks}[1]{leftlbrack,{#1},rightrbrack}
newcommand{dd}{mathrm{d}}
newcommand{ds}[1]{displaystyle{#1}}
newcommand{expo}[1]{,mathrm{e}^{#1},}
newcommand{ic}{mathrm{i}}
newcommand{mc}[1]{mathcal{#1}}
newcommand{mrm}[1]{mathrm{#1}}
newcommand{pars}[1]{left(,{#1},right)}
newcommand{partiald}[3]{frac{partial^{#1} #2}{partial #3^{#1}}}
newcommand{root}[2]{,sqrt[#1]{,{#2},},}
newcommand{totald}[3]{frac{mathrm{d}^{#1} #2}{mathrm{d} #3^{#1}}}
newcommand{verts}[1]{leftvert,{#1},rightvert}$




Stirling Asymptotic:
$ds{N! sim root{2pi}, N^{N + 1/2}expo{-N}}$ as
$ds{verts{N} to infty}$.




begin{align}
left.lim_{n to infty}{Gammapars{n + alpha} over Gammapars{n}n^{alpha}},rightvert_{ alpha in mathbb{C}}
& =
lim_{n to infty}{pars{n + alpha - 1}! over
pars{n - 1}!, n^{alpha}}
\[5mm] & =
lim_{n to infty}{root{2pi}pars{n + alpha - 1}^{n + alpha - 1/2}expo{-pars{n + alpha - 1}} over
bracks{root{2pi}pars{n - 1}^{n - 1/2}expo{-pars{n - 1}}}, n^{alpha}}
\[5mm] & =
lim_{n to infty}{n^{n + alpha - 1/2},
bracks{1 + pars{alpha - 1}/n}^{n + alpha - 1/2},expo{-alpha} over
bracks{n^{n - 1/2}pars{1 - 1/n}^{n - 1/2}}, n^{alpha}}
\[5mm] & =
expo{-alpha}lim_{n to infty}
{bracks{1 + pars{alpha - 1}/n}^{n} over pars{1 - 1/n}^{n}}
\[5mm] & =
expo{-alpha},{expo{alpha - 1} over expo{-1}} = bbx{1}
end{align}






share|cite|improve this answer









$endgroup$





















    3












    $begingroup$

    Real $boldsymbol{alpha}$



    The log-convexity of the Gamma function is shown in this answer.



    Suppose that $0lealphale kinmathbb{Z}$, then using the recurrence relation for $Gamma$,
    $$
    begin{align}
    Gamma(n+alpha)
    &leGamma(n)^{1-alpha/k},Gamma(n+k)^{alpha/k}\
    &leGamma(n)^{1-alpha/k}left(Gamma(n),(n+k)^kright)^{alpha/k}\
    &=Gamma(n),(n+k)^alphatag1
    end{align}
    $$

    and
    $$
    begin{align}
    Gamma(n)
    &leGamma(n+alpha-k)^{alpha/k}Gamma(n+alpha)^{1-alpha/k}\[6pt]
    &leleft(frac{Gamma(n+alpha)}{(n+alpha-k)^k}right)^{alpha/k}Gamma(n+alpha)^{1-alpha/k}\
    &=frac{Gamma(n+alpha)}{(n+alpha-k)^alpha}tag2
    end{align}
    $$

    Then we have
    $$
    left(frac{n+alpha-k}{n}right)^alpha
    lefrac{Gamma(n+alpha)}{Gamma(n),n^alpha}
    leleft(frac{n+k}{n}right)^alphatag3
    $$

    and by the Squeeze Theorem, for $alphage0$,
    $$
    lim_{ntoinfty}frac{Gamma(n+alpha)}{Gamma(n),n^alpha}=1tag4
    $$

    Substituting $nmapsto n-alpha$ in $(3)$, multiplying by $left(frac{n-alpha}nright)^alpha$, and taking reciprocals, we get
    $$
    left(frac{n-k}{n}right)^{-alpha}
    gefrac{Gamma(n-alpha)}{Gamma(n),n^{-alpha}}
    geleft(frac{n-alpha+k}{n}right)^{-alpha}tag5
    $$

    and by the Squeeze Theorem, for $alphage0$,
    $$
    lim_{ntoinfty}frac{Gamma(n-alpha)}{Gamma(n),n^{-alpha}}=1tag6
    $$





    Complex $boldsymbol{alpha}$



    Unfortunately, I have not found a way to make the log-convexity argument that works for $alphainmathbb{R}$ work for $alphain mathbb{C}$. About the best I can see, is to use Stirling's Approximation.
    $$
    Gamma(n)simsqrt{frac{2pi}n}frac{n^n}{e^n}tag7
    $$

    Applying $(7)$ to $Gamma(n+alpha)$ and $Gamma(n)$, we get
    $$
    frac{Gamma(n+alpha)}{Gamma(n),n^alpha}simsqrt{frac{n}{n+alpha}}frac{left(1+fracalpha{n}right)^{n+alpha}}{e^alpha}tag8
    $$

    which yields
    $$
    lim_{ntoinfty}frac{Gamma(n+alpha)}{Gamma(n),n^alpha}=1tag9
    $$






    share|cite|improve this answer











    $endgroup$













    • $begingroup$
      Fyi, the Bohr-Mollerup Wikipedia page proves the real case as a direct corollary.
      $endgroup$
      – adfriedman
      Jan 18 at 2:37










    • $begingroup$
      @adfriedman: that is not surprising since both are using the log-convexity of $Gamma$.
      $endgroup$
      – robjohn
      Jan 18 at 4:15



















    2












    $begingroup$

    As said, Stirling approximation is the key.



    Considering
    $$y=frac{Gamma(n+a)}{Gamma(n),n^a}implies log(y)=log (Gamma (a+n))-log (Gamma (n))-a log(n)$$ use Stirling approximation
    $$log (Gamma (p))=p (log (p)-1)+frac{1}{2} left(log (2 pi
    )-log left({p}right)right)+frac{1}{12 p}-frac{1}{360 p^3}+Oleft(frac{1}{p^5}right)$$
    Just apply it and continue with Taylor series to get
    $$log(y)=frac{(a-1) a}{2 n}-frac{(a-1) a (2 a-1)}{12 n^2}+frac{(a-1)^2 a^2}{12
    n^3}+Oleft(frac{1}{n^4}right)$$
    Continue with Taylor
    $$y=e^{log(y)}=1+frac{(a-1) a}{2 n}+frac{(a-2) (a-1) a (3 a-1)}{24 n^2}+Oleft(frac{1}{n^3}right)$$



    Using it for $n=100$ and $a=5+7i$ the "exact value" would be $(0.798463+ 0.143902 ,i)$ while the above approximation would give $frac{31947}{40000}+frac{1057 }{7500}iapprox (0.798675 +0.140933 i)$.






    share|cite|improve this answer









    $endgroup$













      Your Answer





      StackExchange.ifUsing("editor", function () {
      return StackExchange.using("mathjaxEditing", function () {
      StackExchange.MarkdownEditor.creationCallbacks.add(function (editor, postfix) {
      StackExchange.mathjaxEditing.prepareWmdForMathJax(editor, postfix, [["$", "$"], ["\\(","\\)"]]);
      });
      });
      }, "mathjax-editing");

      StackExchange.ready(function() {
      var channelOptions = {
      tags: "".split(" "),
      id: "69"
      };
      initTagRenderer("".split(" "), "".split(" "), channelOptions);

      StackExchange.using("externalEditor", function() {
      // Have to fire editor after snippets, if snippets enabled
      if (StackExchange.settings.snippets.snippetsEnabled) {
      StackExchange.using("snippets", function() {
      createEditor();
      });
      }
      else {
      createEditor();
      }
      });

      function createEditor() {
      StackExchange.prepareEditor({
      heartbeatType: 'answer',
      autoActivateHeartbeat: false,
      convertImagesToLinks: true,
      noModals: true,
      showLowRepImageUploadWarning: true,
      reputationToPostImages: 10,
      bindNavPrevention: true,
      postfix: "",
      imageUploader: {
      brandingHtml: "Powered by u003ca class="icon-imgur-white" href="https://imgur.com/"u003eu003c/au003e",
      contentPolicyHtml: "User contributions licensed under u003ca href="https://creativecommons.org/licenses/by-sa/3.0/"u003ecc by-sa 3.0 with attribution requiredu003c/au003e u003ca href="https://stackoverflow.com/legal/content-policy"u003e(content policy)u003c/au003e",
      allowUrls: true
      },
      noCode: true, onDemand: true,
      discardSelector: ".discard-answer"
      ,immediatelyShowMarkdownHelp:true
      });


      }
      });














      draft saved

      draft discarded


















      StackExchange.ready(
      function () {
      StackExchange.openid.initPostLogin('.new-post-login', 'https%3a%2f%2fmath.stackexchange.com%2fquestions%2f3076357%2fasymptotic-approximation-regarding-the-gamma-function-gamma%23new-answer', 'question_page');
      }
      );

      Post as a guest















      Required, but never shown

























      4 Answers
      4






      active

      oldest

      votes








      4 Answers
      4






      active

      oldest

      votes









      active

      oldest

      votes






      active

      oldest

      votes









      5












      $begingroup$

      The most usual derivation of this would involve the Stirling-Laplace asymptotic for $Gamma(s)$. I'm mildly surprised that this wasn't explicitly worked out in Wiki, or some other easily accessible places on-line.



      In fact, a much simpler approach obtains (a stronger version of) this asymptotic via "Watson's Lemma", which is itself easy to completely prove from simple things, going back over 100 years. In various places in the literature, the lemma is in fact called "the oft-reproven Watson's lemma". :)



      The case you mention is a simple corollary of the very first example I wrote out in some notes on asymptotic expansions: http://www.math.umn.edu/~garrett/m/mfms/notes_2013-14/02d_asymptotics_of_integrals.pdf






      share|cite|improve this answer











      $endgroup$









      • 1




        $begingroup$
        Thank you for you answer. By the way, your link doesn't work for me for some reason.
        $endgroup$
        – BigbearZzz
        Jan 16 at 22:53












      • $begingroup$
        Oop, let me try to repair the link...
        $endgroup$
        – paul garrett
        Jan 16 at 23:03










      • $begingroup$
        I think it works now... :)
        $endgroup$
        – paul garrett
        Jan 16 at 23:04










      • $begingroup$
        Now it also works perfectly for me. Thanks!
        $endgroup$
        – BigbearZzz
        Jan 16 at 23:16
















      5












      $begingroup$

      The most usual derivation of this would involve the Stirling-Laplace asymptotic for $Gamma(s)$. I'm mildly surprised that this wasn't explicitly worked out in Wiki, or some other easily accessible places on-line.



      In fact, a much simpler approach obtains (a stronger version of) this asymptotic via "Watson's Lemma", which is itself easy to completely prove from simple things, going back over 100 years. In various places in the literature, the lemma is in fact called "the oft-reproven Watson's lemma". :)



      The case you mention is a simple corollary of the very first example I wrote out in some notes on asymptotic expansions: http://www.math.umn.edu/~garrett/m/mfms/notes_2013-14/02d_asymptotics_of_integrals.pdf






      share|cite|improve this answer











      $endgroup$









      • 1




        $begingroup$
        Thank you for you answer. By the way, your link doesn't work for me for some reason.
        $endgroup$
        – BigbearZzz
        Jan 16 at 22:53












      • $begingroup$
        Oop, let me try to repair the link...
        $endgroup$
        – paul garrett
        Jan 16 at 23:03










      • $begingroup$
        I think it works now... :)
        $endgroup$
        – paul garrett
        Jan 16 at 23:04










      • $begingroup$
        Now it also works perfectly for me. Thanks!
        $endgroup$
        – BigbearZzz
        Jan 16 at 23:16














      5












      5








      5





      $begingroup$

      The most usual derivation of this would involve the Stirling-Laplace asymptotic for $Gamma(s)$. I'm mildly surprised that this wasn't explicitly worked out in Wiki, or some other easily accessible places on-line.



      In fact, a much simpler approach obtains (a stronger version of) this asymptotic via "Watson's Lemma", which is itself easy to completely prove from simple things, going back over 100 years. In various places in the literature, the lemma is in fact called "the oft-reproven Watson's lemma". :)



      The case you mention is a simple corollary of the very first example I wrote out in some notes on asymptotic expansions: http://www.math.umn.edu/~garrett/m/mfms/notes_2013-14/02d_asymptotics_of_integrals.pdf






      share|cite|improve this answer











      $endgroup$



      The most usual derivation of this would involve the Stirling-Laplace asymptotic for $Gamma(s)$. I'm mildly surprised that this wasn't explicitly worked out in Wiki, or some other easily accessible places on-line.



      In fact, a much simpler approach obtains (a stronger version of) this asymptotic via "Watson's Lemma", which is itself easy to completely prove from simple things, going back over 100 years. In various places in the literature, the lemma is in fact called "the oft-reproven Watson's lemma". :)



      The case you mention is a simple corollary of the very first example I wrote out in some notes on asymptotic expansions: http://www.math.umn.edu/~garrett/m/mfms/notes_2013-14/02d_asymptotics_of_integrals.pdf







      share|cite|improve this answer














      share|cite|improve this answer



      share|cite|improve this answer








      edited Jan 16 at 23:04

























      answered Jan 16 at 22:51









      paul garrettpaul garrett

      31.9k362118




      31.9k362118








      • 1




        $begingroup$
        Thank you for you answer. By the way, your link doesn't work for me for some reason.
        $endgroup$
        – BigbearZzz
        Jan 16 at 22:53












      • $begingroup$
        Oop, let me try to repair the link...
        $endgroup$
        – paul garrett
        Jan 16 at 23:03










      • $begingroup$
        I think it works now... :)
        $endgroup$
        – paul garrett
        Jan 16 at 23:04










      • $begingroup$
        Now it also works perfectly for me. Thanks!
        $endgroup$
        – BigbearZzz
        Jan 16 at 23:16














      • 1




        $begingroup$
        Thank you for you answer. By the way, your link doesn't work for me for some reason.
        $endgroup$
        – BigbearZzz
        Jan 16 at 22:53












      • $begingroup$
        Oop, let me try to repair the link...
        $endgroup$
        – paul garrett
        Jan 16 at 23:03










      • $begingroup$
        I think it works now... :)
        $endgroup$
        – paul garrett
        Jan 16 at 23:04










      • $begingroup$
        Now it also works perfectly for me. Thanks!
        $endgroup$
        – BigbearZzz
        Jan 16 at 23:16








      1




      1




      $begingroup$
      Thank you for you answer. By the way, your link doesn't work for me for some reason.
      $endgroup$
      – BigbearZzz
      Jan 16 at 22:53






      $begingroup$
      Thank you for you answer. By the way, your link doesn't work for me for some reason.
      $endgroup$
      – BigbearZzz
      Jan 16 at 22:53














      $begingroup$
      Oop, let me try to repair the link...
      $endgroup$
      – paul garrett
      Jan 16 at 23:03




      $begingroup$
      Oop, let me try to repair the link...
      $endgroup$
      – paul garrett
      Jan 16 at 23:03












      $begingroup$
      I think it works now... :)
      $endgroup$
      – paul garrett
      Jan 16 at 23:04




      $begingroup$
      I think it works now... :)
      $endgroup$
      – paul garrett
      Jan 16 at 23:04












      $begingroup$
      Now it also works perfectly for me. Thanks!
      $endgroup$
      – BigbearZzz
      Jan 16 at 23:16




      $begingroup$
      Now it also works perfectly for me. Thanks!
      $endgroup$
      – BigbearZzz
      Jan 16 at 23:16











      4












      $begingroup$

      $newcommand{bbx}[1]{,bbox[15px,border:1px groove navy]{displaystyle{#1}},}
      newcommand{braces}[1]{leftlbrace,{#1},rightrbrace}
      newcommand{bracks}[1]{leftlbrack,{#1},rightrbrack}
      newcommand{dd}{mathrm{d}}
      newcommand{ds}[1]{displaystyle{#1}}
      newcommand{expo}[1]{,mathrm{e}^{#1},}
      newcommand{ic}{mathrm{i}}
      newcommand{mc}[1]{mathcal{#1}}
      newcommand{mrm}[1]{mathrm{#1}}
      newcommand{pars}[1]{left(,{#1},right)}
      newcommand{partiald}[3]{frac{partial^{#1} #2}{partial #3^{#1}}}
      newcommand{root}[2]{,sqrt[#1]{,{#2},},}
      newcommand{totald}[3]{frac{mathrm{d}^{#1} #2}{mathrm{d} #3^{#1}}}
      newcommand{verts}[1]{leftvert,{#1},rightvert}$




      Stirling Asymptotic:
      $ds{N! sim root{2pi}, N^{N + 1/2}expo{-N}}$ as
      $ds{verts{N} to infty}$.




      begin{align}
      left.lim_{n to infty}{Gammapars{n + alpha} over Gammapars{n}n^{alpha}},rightvert_{ alpha in mathbb{C}}
      & =
      lim_{n to infty}{pars{n + alpha - 1}! over
      pars{n - 1}!, n^{alpha}}
      \[5mm] & =
      lim_{n to infty}{root{2pi}pars{n + alpha - 1}^{n + alpha - 1/2}expo{-pars{n + alpha - 1}} over
      bracks{root{2pi}pars{n - 1}^{n - 1/2}expo{-pars{n - 1}}}, n^{alpha}}
      \[5mm] & =
      lim_{n to infty}{n^{n + alpha - 1/2},
      bracks{1 + pars{alpha - 1}/n}^{n + alpha - 1/2},expo{-alpha} over
      bracks{n^{n - 1/2}pars{1 - 1/n}^{n - 1/2}}, n^{alpha}}
      \[5mm] & =
      expo{-alpha}lim_{n to infty}
      {bracks{1 + pars{alpha - 1}/n}^{n} over pars{1 - 1/n}^{n}}
      \[5mm] & =
      expo{-alpha},{expo{alpha - 1} over expo{-1}} = bbx{1}
      end{align}






      share|cite|improve this answer









      $endgroup$


















        4












        $begingroup$

        $newcommand{bbx}[1]{,bbox[15px,border:1px groove navy]{displaystyle{#1}},}
        newcommand{braces}[1]{leftlbrace,{#1},rightrbrace}
        newcommand{bracks}[1]{leftlbrack,{#1},rightrbrack}
        newcommand{dd}{mathrm{d}}
        newcommand{ds}[1]{displaystyle{#1}}
        newcommand{expo}[1]{,mathrm{e}^{#1},}
        newcommand{ic}{mathrm{i}}
        newcommand{mc}[1]{mathcal{#1}}
        newcommand{mrm}[1]{mathrm{#1}}
        newcommand{pars}[1]{left(,{#1},right)}
        newcommand{partiald}[3]{frac{partial^{#1} #2}{partial #3^{#1}}}
        newcommand{root}[2]{,sqrt[#1]{,{#2},},}
        newcommand{totald}[3]{frac{mathrm{d}^{#1} #2}{mathrm{d} #3^{#1}}}
        newcommand{verts}[1]{leftvert,{#1},rightvert}$




        Stirling Asymptotic:
        $ds{N! sim root{2pi}, N^{N + 1/2}expo{-N}}$ as
        $ds{verts{N} to infty}$.




        begin{align}
        left.lim_{n to infty}{Gammapars{n + alpha} over Gammapars{n}n^{alpha}},rightvert_{ alpha in mathbb{C}}
        & =
        lim_{n to infty}{pars{n + alpha - 1}! over
        pars{n - 1}!, n^{alpha}}
        \[5mm] & =
        lim_{n to infty}{root{2pi}pars{n + alpha - 1}^{n + alpha - 1/2}expo{-pars{n + alpha - 1}} over
        bracks{root{2pi}pars{n - 1}^{n - 1/2}expo{-pars{n - 1}}}, n^{alpha}}
        \[5mm] & =
        lim_{n to infty}{n^{n + alpha - 1/2},
        bracks{1 + pars{alpha - 1}/n}^{n + alpha - 1/2},expo{-alpha} over
        bracks{n^{n - 1/2}pars{1 - 1/n}^{n - 1/2}}, n^{alpha}}
        \[5mm] & =
        expo{-alpha}lim_{n to infty}
        {bracks{1 + pars{alpha - 1}/n}^{n} over pars{1 - 1/n}^{n}}
        \[5mm] & =
        expo{-alpha},{expo{alpha - 1} over expo{-1}} = bbx{1}
        end{align}






        share|cite|improve this answer









        $endgroup$
















          4












          4








          4





          $begingroup$

          $newcommand{bbx}[1]{,bbox[15px,border:1px groove navy]{displaystyle{#1}},}
          newcommand{braces}[1]{leftlbrace,{#1},rightrbrace}
          newcommand{bracks}[1]{leftlbrack,{#1},rightrbrack}
          newcommand{dd}{mathrm{d}}
          newcommand{ds}[1]{displaystyle{#1}}
          newcommand{expo}[1]{,mathrm{e}^{#1},}
          newcommand{ic}{mathrm{i}}
          newcommand{mc}[1]{mathcal{#1}}
          newcommand{mrm}[1]{mathrm{#1}}
          newcommand{pars}[1]{left(,{#1},right)}
          newcommand{partiald}[3]{frac{partial^{#1} #2}{partial #3^{#1}}}
          newcommand{root}[2]{,sqrt[#1]{,{#2},},}
          newcommand{totald}[3]{frac{mathrm{d}^{#1} #2}{mathrm{d} #3^{#1}}}
          newcommand{verts}[1]{leftvert,{#1},rightvert}$




          Stirling Asymptotic:
          $ds{N! sim root{2pi}, N^{N + 1/2}expo{-N}}$ as
          $ds{verts{N} to infty}$.




          begin{align}
          left.lim_{n to infty}{Gammapars{n + alpha} over Gammapars{n}n^{alpha}},rightvert_{ alpha in mathbb{C}}
          & =
          lim_{n to infty}{pars{n + alpha - 1}! over
          pars{n - 1}!, n^{alpha}}
          \[5mm] & =
          lim_{n to infty}{root{2pi}pars{n + alpha - 1}^{n + alpha - 1/2}expo{-pars{n + alpha - 1}} over
          bracks{root{2pi}pars{n - 1}^{n - 1/2}expo{-pars{n - 1}}}, n^{alpha}}
          \[5mm] & =
          lim_{n to infty}{n^{n + alpha - 1/2},
          bracks{1 + pars{alpha - 1}/n}^{n + alpha - 1/2},expo{-alpha} over
          bracks{n^{n - 1/2}pars{1 - 1/n}^{n - 1/2}}, n^{alpha}}
          \[5mm] & =
          expo{-alpha}lim_{n to infty}
          {bracks{1 + pars{alpha - 1}/n}^{n} over pars{1 - 1/n}^{n}}
          \[5mm] & =
          expo{-alpha},{expo{alpha - 1} over expo{-1}} = bbx{1}
          end{align}






          share|cite|improve this answer









          $endgroup$



          $newcommand{bbx}[1]{,bbox[15px,border:1px groove navy]{displaystyle{#1}},}
          newcommand{braces}[1]{leftlbrace,{#1},rightrbrace}
          newcommand{bracks}[1]{leftlbrack,{#1},rightrbrack}
          newcommand{dd}{mathrm{d}}
          newcommand{ds}[1]{displaystyle{#1}}
          newcommand{expo}[1]{,mathrm{e}^{#1},}
          newcommand{ic}{mathrm{i}}
          newcommand{mc}[1]{mathcal{#1}}
          newcommand{mrm}[1]{mathrm{#1}}
          newcommand{pars}[1]{left(,{#1},right)}
          newcommand{partiald}[3]{frac{partial^{#1} #2}{partial #3^{#1}}}
          newcommand{root}[2]{,sqrt[#1]{,{#2},},}
          newcommand{totald}[3]{frac{mathrm{d}^{#1} #2}{mathrm{d} #3^{#1}}}
          newcommand{verts}[1]{leftvert,{#1},rightvert}$




          Stirling Asymptotic:
          $ds{N! sim root{2pi}, N^{N + 1/2}expo{-N}}$ as
          $ds{verts{N} to infty}$.




          begin{align}
          left.lim_{n to infty}{Gammapars{n + alpha} over Gammapars{n}n^{alpha}},rightvert_{ alpha in mathbb{C}}
          & =
          lim_{n to infty}{pars{n + alpha - 1}! over
          pars{n - 1}!, n^{alpha}}
          \[5mm] & =
          lim_{n to infty}{root{2pi}pars{n + alpha - 1}^{n + alpha - 1/2}expo{-pars{n + alpha - 1}} over
          bracks{root{2pi}pars{n - 1}^{n - 1/2}expo{-pars{n - 1}}}, n^{alpha}}
          \[5mm] & =
          lim_{n to infty}{n^{n + alpha - 1/2},
          bracks{1 + pars{alpha - 1}/n}^{n + alpha - 1/2},expo{-alpha} over
          bracks{n^{n - 1/2}pars{1 - 1/n}^{n - 1/2}}, n^{alpha}}
          \[5mm] & =
          expo{-alpha}lim_{n to infty}
          {bracks{1 + pars{alpha - 1}/n}^{n} over pars{1 - 1/n}^{n}}
          \[5mm] & =
          expo{-alpha},{expo{alpha - 1} over expo{-1}} = bbx{1}
          end{align}







          share|cite|improve this answer












          share|cite|improve this answer



          share|cite|improve this answer










          answered Jan 17 at 4:54









          Felix MarinFelix Marin

          67.9k7107142




          67.9k7107142























              3












              $begingroup$

              Real $boldsymbol{alpha}$



              The log-convexity of the Gamma function is shown in this answer.



              Suppose that $0lealphale kinmathbb{Z}$, then using the recurrence relation for $Gamma$,
              $$
              begin{align}
              Gamma(n+alpha)
              &leGamma(n)^{1-alpha/k},Gamma(n+k)^{alpha/k}\
              &leGamma(n)^{1-alpha/k}left(Gamma(n),(n+k)^kright)^{alpha/k}\
              &=Gamma(n),(n+k)^alphatag1
              end{align}
              $$

              and
              $$
              begin{align}
              Gamma(n)
              &leGamma(n+alpha-k)^{alpha/k}Gamma(n+alpha)^{1-alpha/k}\[6pt]
              &leleft(frac{Gamma(n+alpha)}{(n+alpha-k)^k}right)^{alpha/k}Gamma(n+alpha)^{1-alpha/k}\
              &=frac{Gamma(n+alpha)}{(n+alpha-k)^alpha}tag2
              end{align}
              $$

              Then we have
              $$
              left(frac{n+alpha-k}{n}right)^alpha
              lefrac{Gamma(n+alpha)}{Gamma(n),n^alpha}
              leleft(frac{n+k}{n}right)^alphatag3
              $$

              and by the Squeeze Theorem, for $alphage0$,
              $$
              lim_{ntoinfty}frac{Gamma(n+alpha)}{Gamma(n),n^alpha}=1tag4
              $$

              Substituting $nmapsto n-alpha$ in $(3)$, multiplying by $left(frac{n-alpha}nright)^alpha$, and taking reciprocals, we get
              $$
              left(frac{n-k}{n}right)^{-alpha}
              gefrac{Gamma(n-alpha)}{Gamma(n),n^{-alpha}}
              geleft(frac{n-alpha+k}{n}right)^{-alpha}tag5
              $$

              and by the Squeeze Theorem, for $alphage0$,
              $$
              lim_{ntoinfty}frac{Gamma(n-alpha)}{Gamma(n),n^{-alpha}}=1tag6
              $$





              Complex $boldsymbol{alpha}$



              Unfortunately, I have not found a way to make the log-convexity argument that works for $alphainmathbb{R}$ work for $alphain mathbb{C}$. About the best I can see, is to use Stirling's Approximation.
              $$
              Gamma(n)simsqrt{frac{2pi}n}frac{n^n}{e^n}tag7
              $$

              Applying $(7)$ to $Gamma(n+alpha)$ and $Gamma(n)$, we get
              $$
              frac{Gamma(n+alpha)}{Gamma(n),n^alpha}simsqrt{frac{n}{n+alpha}}frac{left(1+fracalpha{n}right)^{n+alpha}}{e^alpha}tag8
              $$

              which yields
              $$
              lim_{ntoinfty}frac{Gamma(n+alpha)}{Gamma(n),n^alpha}=1tag9
              $$






              share|cite|improve this answer











              $endgroup$













              • $begingroup$
                Fyi, the Bohr-Mollerup Wikipedia page proves the real case as a direct corollary.
                $endgroup$
                – adfriedman
                Jan 18 at 2:37










              • $begingroup$
                @adfriedman: that is not surprising since both are using the log-convexity of $Gamma$.
                $endgroup$
                – robjohn
                Jan 18 at 4:15
















              3












              $begingroup$

              Real $boldsymbol{alpha}$



              The log-convexity of the Gamma function is shown in this answer.



              Suppose that $0lealphale kinmathbb{Z}$, then using the recurrence relation for $Gamma$,
              $$
              begin{align}
              Gamma(n+alpha)
              &leGamma(n)^{1-alpha/k},Gamma(n+k)^{alpha/k}\
              &leGamma(n)^{1-alpha/k}left(Gamma(n),(n+k)^kright)^{alpha/k}\
              &=Gamma(n),(n+k)^alphatag1
              end{align}
              $$

              and
              $$
              begin{align}
              Gamma(n)
              &leGamma(n+alpha-k)^{alpha/k}Gamma(n+alpha)^{1-alpha/k}\[6pt]
              &leleft(frac{Gamma(n+alpha)}{(n+alpha-k)^k}right)^{alpha/k}Gamma(n+alpha)^{1-alpha/k}\
              &=frac{Gamma(n+alpha)}{(n+alpha-k)^alpha}tag2
              end{align}
              $$

              Then we have
              $$
              left(frac{n+alpha-k}{n}right)^alpha
              lefrac{Gamma(n+alpha)}{Gamma(n),n^alpha}
              leleft(frac{n+k}{n}right)^alphatag3
              $$

              and by the Squeeze Theorem, for $alphage0$,
              $$
              lim_{ntoinfty}frac{Gamma(n+alpha)}{Gamma(n),n^alpha}=1tag4
              $$

              Substituting $nmapsto n-alpha$ in $(3)$, multiplying by $left(frac{n-alpha}nright)^alpha$, and taking reciprocals, we get
              $$
              left(frac{n-k}{n}right)^{-alpha}
              gefrac{Gamma(n-alpha)}{Gamma(n),n^{-alpha}}
              geleft(frac{n-alpha+k}{n}right)^{-alpha}tag5
              $$

              and by the Squeeze Theorem, for $alphage0$,
              $$
              lim_{ntoinfty}frac{Gamma(n-alpha)}{Gamma(n),n^{-alpha}}=1tag6
              $$





              Complex $boldsymbol{alpha}$



              Unfortunately, I have not found a way to make the log-convexity argument that works for $alphainmathbb{R}$ work for $alphain mathbb{C}$. About the best I can see, is to use Stirling's Approximation.
              $$
              Gamma(n)simsqrt{frac{2pi}n}frac{n^n}{e^n}tag7
              $$

              Applying $(7)$ to $Gamma(n+alpha)$ and $Gamma(n)$, we get
              $$
              frac{Gamma(n+alpha)}{Gamma(n),n^alpha}simsqrt{frac{n}{n+alpha}}frac{left(1+fracalpha{n}right)^{n+alpha}}{e^alpha}tag8
              $$

              which yields
              $$
              lim_{ntoinfty}frac{Gamma(n+alpha)}{Gamma(n),n^alpha}=1tag9
              $$






              share|cite|improve this answer











              $endgroup$













              • $begingroup$
                Fyi, the Bohr-Mollerup Wikipedia page proves the real case as a direct corollary.
                $endgroup$
                – adfriedman
                Jan 18 at 2:37










              • $begingroup$
                @adfriedman: that is not surprising since both are using the log-convexity of $Gamma$.
                $endgroup$
                – robjohn
                Jan 18 at 4:15














              3












              3








              3





              $begingroup$

              Real $boldsymbol{alpha}$



              The log-convexity of the Gamma function is shown in this answer.



              Suppose that $0lealphale kinmathbb{Z}$, then using the recurrence relation for $Gamma$,
              $$
              begin{align}
              Gamma(n+alpha)
              &leGamma(n)^{1-alpha/k},Gamma(n+k)^{alpha/k}\
              &leGamma(n)^{1-alpha/k}left(Gamma(n),(n+k)^kright)^{alpha/k}\
              &=Gamma(n),(n+k)^alphatag1
              end{align}
              $$

              and
              $$
              begin{align}
              Gamma(n)
              &leGamma(n+alpha-k)^{alpha/k}Gamma(n+alpha)^{1-alpha/k}\[6pt]
              &leleft(frac{Gamma(n+alpha)}{(n+alpha-k)^k}right)^{alpha/k}Gamma(n+alpha)^{1-alpha/k}\
              &=frac{Gamma(n+alpha)}{(n+alpha-k)^alpha}tag2
              end{align}
              $$

              Then we have
              $$
              left(frac{n+alpha-k}{n}right)^alpha
              lefrac{Gamma(n+alpha)}{Gamma(n),n^alpha}
              leleft(frac{n+k}{n}right)^alphatag3
              $$

              and by the Squeeze Theorem, for $alphage0$,
              $$
              lim_{ntoinfty}frac{Gamma(n+alpha)}{Gamma(n),n^alpha}=1tag4
              $$

              Substituting $nmapsto n-alpha$ in $(3)$, multiplying by $left(frac{n-alpha}nright)^alpha$, and taking reciprocals, we get
              $$
              left(frac{n-k}{n}right)^{-alpha}
              gefrac{Gamma(n-alpha)}{Gamma(n),n^{-alpha}}
              geleft(frac{n-alpha+k}{n}right)^{-alpha}tag5
              $$

              and by the Squeeze Theorem, for $alphage0$,
              $$
              lim_{ntoinfty}frac{Gamma(n-alpha)}{Gamma(n),n^{-alpha}}=1tag6
              $$





              Complex $boldsymbol{alpha}$



              Unfortunately, I have not found a way to make the log-convexity argument that works for $alphainmathbb{R}$ work for $alphain mathbb{C}$. About the best I can see, is to use Stirling's Approximation.
              $$
              Gamma(n)simsqrt{frac{2pi}n}frac{n^n}{e^n}tag7
              $$

              Applying $(7)$ to $Gamma(n+alpha)$ and $Gamma(n)$, we get
              $$
              frac{Gamma(n+alpha)}{Gamma(n),n^alpha}simsqrt{frac{n}{n+alpha}}frac{left(1+fracalpha{n}right)^{n+alpha}}{e^alpha}tag8
              $$

              which yields
              $$
              lim_{ntoinfty}frac{Gamma(n+alpha)}{Gamma(n),n^alpha}=1tag9
              $$






              share|cite|improve this answer











              $endgroup$



              Real $boldsymbol{alpha}$



              The log-convexity of the Gamma function is shown in this answer.



              Suppose that $0lealphale kinmathbb{Z}$, then using the recurrence relation for $Gamma$,
              $$
              begin{align}
              Gamma(n+alpha)
              &leGamma(n)^{1-alpha/k},Gamma(n+k)^{alpha/k}\
              &leGamma(n)^{1-alpha/k}left(Gamma(n),(n+k)^kright)^{alpha/k}\
              &=Gamma(n),(n+k)^alphatag1
              end{align}
              $$

              and
              $$
              begin{align}
              Gamma(n)
              &leGamma(n+alpha-k)^{alpha/k}Gamma(n+alpha)^{1-alpha/k}\[6pt]
              &leleft(frac{Gamma(n+alpha)}{(n+alpha-k)^k}right)^{alpha/k}Gamma(n+alpha)^{1-alpha/k}\
              &=frac{Gamma(n+alpha)}{(n+alpha-k)^alpha}tag2
              end{align}
              $$

              Then we have
              $$
              left(frac{n+alpha-k}{n}right)^alpha
              lefrac{Gamma(n+alpha)}{Gamma(n),n^alpha}
              leleft(frac{n+k}{n}right)^alphatag3
              $$

              and by the Squeeze Theorem, for $alphage0$,
              $$
              lim_{ntoinfty}frac{Gamma(n+alpha)}{Gamma(n),n^alpha}=1tag4
              $$

              Substituting $nmapsto n-alpha$ in $(3)$, multiplying by $left(frac{n-alpha}nright)^alpha$, and taking reciprocals, we get
              $$
              left(frac{n-k}{n}right)^{-alpha}
              gefrac{Gamma(n-alpha)}{Gamma(n),n^{-alpha}}
              geleft(frac{n-alpha+k}{n}right)^{-alpha}tag5
              $$

              and by the Squeeze Theorem, for $alphage0$,
              $$
              lim_{ntoinfty}frac{Gamma(n-alpha)}{Gamma(n),n^{-alpha}}=1tag6
              $$





              Complex $boldsymbol{alpha}$



              Unfortunately, I have not found a way to make the log-convexity argument that works for $alphainmathbb{R}$ work for $alphain mathbb{C}$. About the best I can see, is to use Stirling's Approximation.
              $$
              Gamma(n)simsqrt{frac{2pi}n}frac{n^n}{e^n}tag7
              $$

              Applying $(7)$ to $Gamma(n+alpha)$ and $Gamma(n)$, we get
              $$
              frac{Gamma(n+alpha)}{Gamma(n),n^alpha}simsqrt{frac{n}{n+alpha}}frac{left(1+fracalpha{n}right)^{n+alpha}}{e^alpha}tag8
              $$

              which yields
              $$
              lim_{ntoinfty}frac{Gamma(n+alpha)}{Gamma(n),n^alpha}=1tag9
              $$







              share|cite|improve this answer














              share|cite|improve this answer



              share|cite|improve this answer








              edited Jan 18 at 16:17

























              answered Jan 17 at 5:59









              robjohnrobjohn

              267k27308632




              267k27308632












              • $begingroup$
                Fyi, the Bohr-Mollerup Wikipedia page proves the real case as a direct corollary.
                $endgroup$
                – adfriedman
                Jan 18 at 2:37










              • $begingroup$
                @adfriedman: that is not surprising since both are using the log-convexity of $Gamma$.
                $endgroup$
                – robjohn
                Jan 18 at 4:15


















              • $begingroup$
                Fyi, the Bohr-Mollerup Wikipedia page proves the real case as a direct corollary.
                $endgroup$
                – adfriedman
                Jan 18 at 2:37










              • $begingroup$
                @adfriedman: that is not surprising since both are using the log-convexity of $Gamma$.
                $endgroup$
                – robjohn
                Jan 18 at 4:15
















              $begingroup$
              Fyi, the Bohr-Mollerup Wikipedia page proves the real case as a direct corollary.
              $endgroup$
              – adfriedman
              Jan 18 at 2:37




              $begingroup$
              Fyi, the Bohr-Mollerup Wikipedia page proves the real case as a direct corollary.
              $endgroup$
              – adfriedman
              Jan 18 at 2:37












              $begingroup$
              @adfriedman: that is not surprising since both are using the log-convexity of $Gamma$.
              $endgroup$
              – robjohn
              Jan 18 at 4:15




              $begingroup$
              @adfriedman: that is not surprising since both are using the log-convexity of $Gamma$.
              $endgroup$
              – robjohn
              Jan 18 at 4:15











              2












              $begingroup$

              As said, Stirling approximation is the key.



              Considering
              $$y=frac{Gamma(n+a)}{Gamma(n),n^a}implies log(y)=log (Gamma (a+n))-log (Gamma (n))-a log(n)$$ use Stirling approximation
              $$log (Gamma (p))=p (log (p)-1)+frac{1}{2} left(log (2 pi
              )-log left({p}right)right)+frac{1}{12 p}-frac{1}{360 p^3}+Oleft(frac{1}{p^5}right)$$
              Just apply it and continue with Taylor series to get
              $$log(y)=frac{(a-1) a}{2 n}-frac{(a-1) a (2 a-1)}{12 n^2}+frac{(a-1)^2 a^2}{12
              n^3}+Oleft(frac{1}{n^4}right)$$
              Continue with Taylor
              $$y=e^{log(y)}=1+frac{(a-1) a}{2 n}+frac{(a-2) (a-1) a (3 a-1)}{24 n^2}+Oleft(frac{1}{n^3}right)$$



              Using it for $n=100$ and $a=5+7i$ the "exact value" would be $(0.798463+ 0.143902 ,i)$ while the above approximation would give $frac{31947}{40000}+frac{1057 }{7500}iapprox (0.798675 +0.140933 i)$.






              share|cite|improve this answer









              $endgroup$


















                2












                $begingroup$

                As said, Stirling approximation is the key.



                Considering
                $$y=frac{Gamma(n+a)}{Gamma(n),n^a}implies log(y)=log (Gamma (a+n))-log (Gamma (n))-a log(n)$$ use Stirling approximation
                $$log (Gamma (p))=p (log (p)-1)+frac{1}{2} left(log (2 pi
                )-log left({p}right)right)+frac{1}{12 p}-frac{1}{360 p^3}+Oleft(frac{1}{p^5}right)$$
                Just apply it and continue with Taylor series to get
                $$log(y)=frac{(a-1) a}{2 n}-frac{(a-1) a (2 a-1)}{12 n^2}+frac{(a-1)^2 a^2}{12
                n^3}+Oleft(frac{1}{n^4}right)$$
                Continue with Taylor
                $$y=e^{log(y)}=1+frac{(a-1) a}{2 n}+frac{(a-2) (a-1) a (3 a-1)}{24 n^2}+Oleft(frac{1}{n^3}right)$$



                Using it for $n=100$ and $a=5+7i$ the "exact value" would be $(0.798463+ 0.143902 ,i)$ while the above approximation would give $frac{31947}{40000}+frac{1057 }{7500}iapprox (0.798675 +0.140933 i)$.






                share|cite|improve this answer









                $endgroup$
















                  2












                  2








                  2





                  $begingroup$

                  As said, Stirling approximation is the key.



                  Considering
                  $$y=frac{Gamma(n+a)}{Gamma(n),n^a}implies log(y)=log (Gamma (a+n))-log (Gamma (n))-a log(n)$$ use Stirling approximation
                  $$log (Gamma (p))=p (log (p)-1)+frac{1}{2} left(log (2 pi
                  )-log left({p}right)right)+frac{1}{12 p}-frac{1}{360 p^3}+Oleft(frac{1}{p^5}right)$$
                  Just apply it and continue with Taylor series to get
                  $$log(y)=frac{(a-1) a}{2 n}-frac{(a-1) a (2 a-1)}{12 n^2}+frac{(a-1)^2 a^2}{12
                  n^3}+Oleft(frac{1}{n^4}right)$$
                  Continue with Taylor
                  $$y=e^{log(y)}=1+frac{(a-1) a}{2 n}+frac{(a-2) (a-1) a (3 a-1)}{24 n^2}+Oleft(frac{1}{n^3}right)$$



                  Using it for $n=100$ and $a=5+7i$ the "exact value" would be $(0.798463+ 0.143902 ,i)$ while the above approximation would give $frac{31947}{40000}+frac{1057 }{7500}iapprox (0.798675 +0.140933 i)$.






                  share|cite|improve this answer









                  $endgroup$



                  As said, Stirling approximation is the key.



                  Considering
                  $$y=frac{Gamma(n+a)}{Gamma(n),n^a}implies log(y)=log (Gamma (a+n))-log (Gamma (n))-a log(n)$$ use Stirling approximation
                  $$log (Gamma (p))=p (log (p)-1)+frac{1}{2} left(log (2 pi
                  )-log left({p}right)right)+frac{1}{12 p}-frac{1}{360 p^3}+Oleft(frac{1}{p^5}right)$$
                  Just apply it and continue with Taylor series to get
                  $$log(y)=frac{(a-1) a}{2 n}-frac{(a-1) a (2 a-1)}{12 n^2}+frac{(a-1)^2 a^2}{12
                  n^3}+Oleft(frac{1}{n^4}right)$$
                  Continue with Taylor
                  $$y=e^{log(y)}=1+frac{(a-1) a}{2 n}+frac{(a-2) (a-1) a (3 a-1)}{24 n^2}+Oleft(frac{1}{n^3}right)$$



                  Using it for $n=100$ and $a=5+7i$ the "exact value" would be $(0.798463+ 0.143902 ,i)$ while the above approximation would give $frac{31947}{40000}+frac{1057 }{7500}iapprox (0.798675 +0.140933 i)$.







                  share|cite|improve this answer












                  share|cite|improve this answer



                  share|cite|improve this answer










                  answered Jan 17 at 4:36









                  Claude LeiboviciClaude Leibovici

                  121k1157133




                  121k1157133






























                      draft saved

                      draft discarded




















































                      Thanks for contributing an answer to Mathematics Stack Exchange!


                      • Please be sure to answer the question. Provide details and share your research!

                      But avoid



                      • Asking for help, clarification, or responding to other answers.

                      • Making statements based on opinion; back them up with references or personal experience.


                      Use MathJax to format equations. MathJax reference.


                      To learn more, see our tips on writing great answers.




                      draft saved


                      draft discarded














                      StackExchange.ready(
                      function () {
                      StackExchange.openid.initPostLogin('.new-post-login', 'https%3a%2f%2fmath.stackexchange.com%2fquestions%2f3076357%2fasymptotic-approximation-regarding-the-gamma-function-gamma%23new-answer', 'question_page');
                      }
                      );

                      Post as a guest















                      Required, but never shown





















































                      Required, but never shown














                      Required, but never shown












                      Required, but never shown







                      Required, but never shown

































                      Required, but never shown














                      Required, but never shown












                      Required, but never shown







                      Required, but never shown







                      Popular posts from this blog

                      Mario Kart Wii

                      What does “Dominus providebit” mean?

                      Antonio Litta Visconti Arese